1993 AJHSME Problems/Problem 23

Revision as of 16:09, 22 November 2012 by AndrewKwon97 (talk | contribs)

Problem

Five runners, $P$, $Q$, $R$, $S$, $T$, have a race, and $P$ beats $Q$, $P$ beats $R$, $Q$ beats $S$, and $T$ finishes after $P$ and before $Q$. Who could NOT have finished third in the race?

$\text{(A)}\ P\text{ and }Q \qquad \text{(B)}\ P\text{ and }R \qquad \text{(C)}\ P\text{ and }S \qquad \text{(D)}\ P\text{ and }T \qquad \text{(E)}\ P,S\text{ and }T$

Solution

First, note that $P$ must beat $Q$, $R$, $T$, and by transitivity, $S$. Thus $P$ is in first, and not in 3rd. Similarly, $S$ is beaten by $P$, $Q$, and by transitivity, $T$, so $S$ is in fourth or fifth, and not in third. All of the others can be in third, as all of the following sequences show. Each follows all of the assumptions of the problem, and they are in order from first to last: PTQRS PTRQS PRTQS Thus the answer is C.